Prove U (Ai X Bi) [itex]\subseteq[/itex] (U Ai) X (U Bi)

  • Thread starter Thread starter IntroAnalysis
  • Start date Start date
  • Tags Tags
    Ai
Click For Summary
The discussion centers on proving that the union of the Cartesian products of indexed families of sets, U i ∈ I (Ai X Bi), is a subset of the Cartesian product of the unions, (U i ∈ I Ai) X (U i ∈ I Bi). The proof begins by considering an arbitrary element (x, y) in the union of the products, establishing that there exists an index i such that x is in Ai and y is in Bi. This leads to the conclusion that x is in the union of Ai and y is in the union of Bi, confirming the subset relationship. A suggestion is made to explore a counterexample for the reverse inclusion, highlighting the complexity of the topic. The discussion emphasizes the importance of clarity in mathematical proofs, especially in academic settings.
IntroAnalysis
Messages
58
Reaction score
0

Homework Statement


Suppose {Ai l i \inI} and {Bi l i \inI} are indexed families of sets.

Prove that U i \inI(Ai X Bi) \subseteq (Ui \inIAi) X (Ui \inIBi)


2. Relevant\subseteq equations
From How to Prove It, 2nd Edition, Sec. 4.1 #11a)


The Attempt at a Solution



Let (x, y) be arbitrary. Suppose (x, y) \in \bigcupi\inI (Ai X Bi).

Since (x, y) \in\bigcupi\inI(Ai X Bi), there exists an i\inI with x\inAi and y\inBi.

So x \in{xl\existsi\inI(x\inAi)} and
y\in{yl\existsi\inI(y\inBi)}

Therefore, x \in\bigcupi\inI Ai and y\in<br /> \bigcupi\inI Bi.

This is equivalent to (\bigcupi\inI Ai) X (\bigcupi\inI Bi). Hence, \bigcupi\inI (Ai X Bi)\subseteq(Ui\inI Ai) X (\bigcupi\inI Bi).
 
Physics news on Phys.org
That looks very good!

It might be instructive to find a counterexample to the reverse inclusion...
 
Thanks for commenting, I appreciate it. I have a very picky professor.
 
Question: A clock's minute hand has length 4 and its hour hand has length 3. What is the distance between the tips at the moment when it is increasing most rapidly?(Putnam Exam Question) Answer: Making assumption that both the hands moves at constant angular velocities, the answer is ## \sqrt{7} .## But don't you think this assumption is somewhat doubtful and wrong?

Similar threads

  • · Replies 1 ·
Replies
1
Views
6K
  • · Replies 9 ·
Replies
9
Views
2K
  • · Replies 4 ·
Replies
4
Views
3K
Replies
3
Views
2K
  • · Replies 3 ·
Replies
3
Views
2K
Replies
20
Views
4K
Replies
2
Views
1K
  • · Replies 2 ·
Replies
2
Views
2K
  • · Replies 1 ·
Replies
1
Views
2K
  • · Replies 1 ·
Replies
1
Views
2K